Đến nội dung

superpower nội dung

Có 486 mục bởi superpower (Tìm giới hạn từ 08-06-2020)



Sắp theo                Sắp xếp  

#604683 x,y nguyên dương

Đã gửi bởi superpower on 22-12-2015 - 18:51 trong Số học

x2+x$\vdots$xy-1

Đặt $x^2+x= k(xy-1) <=> x^2 + (1-ky)x +k=0  (*)$

$(*)$ có nghiệm nguyên $<=> \triangle = (1-ky)^2 -4k= k^{2}y^{2} -2ky + 1-4k  (**)$ là số chính phương 

TH1: $k=0 => x^2 + x =0 => x=0 hoặc x=-1 $ ( thỏa)

TH2: $k$ khác $0$, xem (**) là phương trình bậc 2 ẩn $y$ là số chính phương thì $\triangle=0 <=> 4k^2 - 4k^2(1-4k)=0 <=> k=0 $ (loại)

Vậy $x=0$ thì y thỏa với mọi $y$ thuộc $R$

       $x=-1$ thì y thỏa với mọi $y$ khác $-1$ ( do mẫu khác $0$) 




#619452 với p là số nguyên tố.đặt n=$\frac{2^{2p}-1}...

Đã gửi bởi superpower on 09-03-2016 - 23:40 trong Số học

vói p là số nguyên tố.đặt n=$\frac{2^{2p}-1}{3}  tìm p nguyên tố thỏa mãn  $2^{n}-2$ ko chia hết cho n

Kí hiệu $[x]$ là mod $x$

Ta có

Giả sử $n$ thỏa: $2^n-2 \vdots n $

Khi đó $2^n \equiv 2 [n] $

Dễ thấy $n$ lẻ

Gọi $q$ là 1 ước nguyên tố của $n$

Khi đó, ta có $2^{n-1} \equiv 1 [q] $

Mặt khác, ta cũng có $2^{q-1} \equiv 1 [q] $

Gọi $h=ord_q 2 $

Do đó $h | n-1 ; h| q-1 ; n=kq $

Do đó $h |2 $

Khi đó $h=1$ hoặc $h=2$

TH1: $h=1$, khi đó, ta có

$2 \equiv 1 [q] $ (vô lý)

Th2: $h=2=> q=3 $

Do đó, để $n$ thỏa thì $n$ không chia hết cho $3$

Làm được tới đây




#619434 với p là số nguyên tố.đặt n=$\frac{2^{2p}-1}...

Đã gửi bởi superpower on 09-03-2016 - 22:22 trong Số học

$\frac{2^{2p}-1}{3}            tìm p           nguyên            tố      thỏa        mãn       2^{n}-2     không      chia   hết      cho     n$

Bạn ghi đề lại đi bạn

Mình đọc không hiểu




#616585 VINACAL 570ES PLUS II thật là khó hiểu?

Đã gửi bởi superpower on 23-02-2016 - 20:18 trong Giải toán bằng máy tính bỏ túi

M

 

Ví dụ nhập vào máy tính 123456789123456789 x 1
Ta được kết quả 1,234567891 x 10^17 (trên màn hình)
Sau đó trừ cho 1,23456789 x 10^ 17
Ta được kq 123456789
Do đó kết quả không có gì thay đổi máy không làm tròn số 9 thành số 0
Nhưng khi tôi dùng tổng xích ma từ 22 + 22^2 + .... + 22^13
Kết quả của tổng này cũng có 18 chữ số và có chữ số tận cùng phải là 2 nhưng trong máy tính lại làm tròn thành 0.
Các m,n nào có sử dụng VINACAL 570es plus II thì thử test dùm
Có m,n nào giải thích dùm?

Máy vinacal chỉ tính chính xác đến 17 chữ số thôi

18 chữ số trở lên thì máy sẽ làm tròn

Đơn giản vậy thôi

Mình nhớ không nhầm thì casio tối đa cũng chỉ 14,15 gì thôi




#628714 USAMO 2016

Đã gửi bởi superpower on 21-04-2016 - 17:09 trong Thi HSG Quốc gia và Quốc tế

 

Ngày 2 (20/04/16)
Bài 4. Tìm tất cả các hàm $f:\mathbb{R} \to \mathbb{R}$ sao cho với mọi số thực $x, y$, $$(f(x) + xy)f(x - 3y) + (f(y) + xy)f(3x - y) = (f(x+y))^{2}$$

Cho mình chỉnh sửa lại câu hàm

Thay $x=y=0 => f(0)=0 $

Thay $x=0 => f(y)f(-y)=f^2(y) => f(y) =0 $ hoặc $f(y) =f(-y) $

TH1: $f(x)=0$ thỏa YCBT

TH2: $f(x)$ không đồng nhất 0; khi đó $f(x)=f(-x) $ suy ra $f$ chẵn

Thay $y=-x => (f(x)-x^2)f(4x)=0 => f(x)=x^2 $

Thử lại thỏa

Nhận xét thì bài hàm này khá dễ 

 




#606573 TÌm $x,y,z\in \mathbb{Z}$ thỏa mãn: $x^2-y...

Đã gửi bởi superpower on 01-01-2016 - 20:17 trong Số học

1/Tìm $a,b,c\in \mathbb{Z}$ thỏa mãn:

                       $a^3(b-c)+b^3(c-a)+c^3(a-b)=1964^{2013}$

 

2/TÌm $x,y,z\in \mathbb{Z}$ thỏa mãn:

                        $x^2-y^2=8^z+10$

Còn bài 1:

Phân tích nhân tử, ta được 

$(a-b)(c-a)(c-b)(a+b+c)= 1964^{2013}$

Tới đây, trâu bò xét TH là ok




#606570 TÌm $x,y,z\in \mathbb{Z}$ thỏa mãn: $x^2-y...

Đã gửi bởi superpower on 01-01-2016 - 20:09 trong Số học

1/Tìm $a,b,c\in \mathbb{Z}$ thỏa mãn:

                       $a^3(b-c)+b^3(c-a)+c^3(a-b)=1964^{2013}$

 

2/TÌm $x,y,z\in \mathbb{Z}$ thỏa mãn:

                        $x^2-y^2=8^z+10$

Xin chém bài 2 trước

Xét $z$ khác $0$

Dễ thấy VT $\equiv VP \equiv  0 $ (mod $2$)

Suy ra $x^2 -y^2=(x-y)(x+y) $ chẵn

Mà do $(x-y) ; (x+y) $ có cùng tính chẵn lẻ => VT $\equiv 0$ ( mod $4$ )

Suy ra VP $\equiv 0 $ ( mod $4$ ) Vô lý do $10$ không chia hết cho $4$

Do đó $z=0$

$=> x^2-y^2 = (x-y)(x+y)=11 => (x=6;y=5); (x=-6;y=-5) ; (x=6;y=-5 ) ; (x=-6;y=5)$




#647398 Trại hè Hùng Vương 2016 Toán 10

Đã gửi bởi superpower on 31-07-2016 - 21:22 trong Thi HSG cấp Tỉnh, Thành phố. Olympic 30-4. Đề thi và kiểm tra đội tuyển các cấp.

Bài 1: Giải phương trình trên tập số thực

$ 4\sqrt{x+1} + 2\sqrt{2x+3} = (x-1)(x^2 -2 ) $ 

 

Mình xin chém bài $1$ trước

ĐK: $x \geq -1 $

Ta có $4(\sqrt{x+1} -2 ) + 2(\sqrt{2x+3} -1 ) = x^3 -x^2 -2x -12 $

          $<=> (x-3)( \frac{4}{\sqrt{x+1} +2 } + \frac{4}{\sqrt{2x+3} +3} ) =(x-3)(x^2+2x+4 ) $

TH1: $x=3 $

TH2: $\frac{4}{\sqrt{x+1} +2 } + \frac{4}{\sqrt{2x+3} +3} =x^2+2x+4 $

Ta có $VP \geq 3 $

$VT \leq \frac{4}{2} + \frac{4}{1+3} =3 $

Do đó $VT=VP <=> x=-1 $

Vậy phương trình có 2 nghiệm $x=-1 ; x=3 $




#647397 Trại hè Hùng Vương 2016 Toán 10

Đã gửi bởi superpower on 31-07-2016 - 21:16 trong Thi HSG cấp Tỉnh, Thành phố. Olympic 30-4. Đề thi và kiểm tra đội tuyển các cấp.

Bài 1: Giải phương trình trên tập số thực

$ 4\sqrt{x+1} + 2\sqrt{2x+3} = (x-1)(x^2 -2 ) $ 

 

Bài 2: Cho tam giác $ABC$ không cân ngoại tiếp đường tròn tâm $I$. Đường tròn $\omega$ tâm $O$ cắt $AI,BI,CI$ lần lượt tại $D,E,F$. Qua $I$ kẻ các đường thẳng song song $BC,CA,AB$ và lần lượt cắt $EF,DF,DE$  tại các điểm $K,L,M$

a/CMR $AK$ tiếp xúc với $\omega$ và $K,L,M$ thẳng hàng

b/Gọi $X$ là giao điểm của $AI$ và $EF$, $Y$ là giao điểm của $BI$ và $DF$, $Z$ là giao điểm của $CI$ và $DE$. Lấy $P$ bất kì trên $BC ( P \neq B,C, P $ không thuộc $AI$ ). CMR đường tròn ngoại tiếp các tam giác $PDX,PEY,PFZ$ cùng đi qua điểm $Q \neq P$ 

 

Bài 3: Cho $a,b,c$ là các số thực thoả $(a+b)(b+c)(c+a) \neq 0 $. CMR: 

$\sum (\frac{a}{a+b} )^2 + \frac{4abc}{(a+b)(b+c)(c+a)} \geq \sum \frac{a}{a+b} - \frac{1}{4} $

 

Bài 4: Cho bảng ô vuông kích thước $10x10$ được chia đều thành $100$ ô vuông, mỗi ô vuông cạnh $1$. Ban đầu người ta tô màu đen cho $k$ ô vuông nào đó trên bảng. Sau đó, nếu ô vuông nào chưa bị tô đen mà nằm cạnh ( có cạnh chung ) với ít nhất 2 ô vuông đen đã tô thì lập tức ô này cũng bị đen. Hãy xác định giá trị nhỏ nhất của $k$ để tới một lúc nào đó, tất cả các ô trên bảng đều bị tô đen 

 

Bài 5: Tìm các số $p,n $ thoả $p$ nguyên tố, $n$ là số nguyên dương sao cho

$p^3 -2p^2 + p +1= 3^n $

 




#647401 Trại hè Hùng Vương 2016 Toán 10

Đã gửi bởi superpower on 31-07-2016 - 21:26 trong Thi HSG cấp Tỉnh, Thành phố. Olympic 30-4. Đề thi và kiểm tra đội tuyển các cấp.

Bài 5: Tìm các số $p,n $ thoả $p$ nguyên tố, $n$ là số nguyên dương sao cho

$p^3 -2p^2 + p +1= 3^n $

 

Khi $p=2 => n=1 $

Giả sử $p \geq 3 $

Ta có $p(p-1)^2 = 3^n -1 $

Khi đó, ta có $n$ phải chẵn vì VT $\vdots 4 $

Do đó $v_2(3^n-1) = v_2(3-1) + v_2(3+1) + v_2(n) -1 = 2+v_2(n) \geq 3  $

Do đó $p-1 $ \vdots 4 $

Suy ra VT $\vdots 16 $

Do đó VP $\vdots 16 => 2+v_2(n) \geq 4 => v_2(n) \geq 2 $

Do đó $n \vdots 4 => n=4k $

Khi đó $p(p-1)^2 = 3^{4k} -1 = 81^k -1 \vdots 80 \vdots 5 $

Suy  ra $p(p-1)^2 \vdots 5 => p=5 $

Khi $p=5 => n=4 $

Do đó có 2 cặp thoả là $(2;1) ; (5;4 ) $ 




#628513 Trong $5$ số nguyên tùy ý bao giờ cũng tìm được $3$ số có...

Đã gửi bởi superpower on 20-04-2016 - 17:35 trong Tổ hợp và rời rạc

 

iúp mình với ...Chứng minh : trong 5 số nguyên tùy ý bao giờ cũng tìm được 3 số có tổng chia hiết cho 3

 

Ta xét 5 số nguyên bất kì

Nếu tồn tại 3 số cùng dư khi chia cho 3 thì ta có đpcm

Do đó, ta chỉ cần xét TH có tối đa 2 số cùng dư khi chia cho 3

Khi đó, gọi A,B,C là số các số chia 3 dư 0,1,2 trong 5 số đó

Ta có $1 \leq |A|, |B|,|C| \leq 2 $

Do đó, ta lấy 1 phần tử của A, 1 của B và 1 của C có đpcm




#650180 Topic: [LTDH] Mỗi ngày hai bất đẳng thức.

Đã gửi bởi superpower on 18-08-2016 - 08:18 trong Bất đẳng thức và cực trị

Lời giải bài 21 và bài 22 các bạn VODANH9X và phamngochung9a đã giải chuẩn rồi, mình xin đề xuất hai bài tiếp theo:

Bài 23: Cho các số thực $a,b,c\in [1,2]$. Tìm GTNN của biểu thức:

$P=\frac{(a+b)^2}{c^2+4(ab+bc+ca)}$

 

Xét hàm số $f(c) = c^2 + 4(ab+bc+ca) $ là hàm đồng biến theo biến $c$

Do đó để $P_{min} $ thì $c $ max

Do đó. ta cần tìm min của biểu thức

$P=\frac{(a+b)^2}{4+4(ab+2a+2b) } $

Mặt khác

Xét hàm số $f(a) = \frac{a^2+2ab+b^2}{(4b+8)a + 4+8b } $

Có $f'(a) = \frac{(4b+8)a^2+ (16b+8)a -4b^3 +8b^2+8b }{[(4b+8)a+4+8b]^2}$

Mà ta có $-4b^3+8b^2+8b > 0 $ (do $b \in [1;2] $)

Do đó $f'(a) >0 $

Tương tự dễ chứng minh $f'(b) >0 $

Mà $a,b \geq 1 => f(a,b) \geq f(1,1) = \frac{1}{6} $

Dấu bằng xảy ra khi $a=b=1; c=2 $ 




#607934 TOPIC ĐỀ THI HSG LỚP 9

Đã gửi bởi superpower on 08-01-2016 - 12:09 trong Tài liệu - Đề thi

 

ĐỀ SỐ 1

 

 

 

Câu 5:

 

a)Cho $a,b,c$ là các số thực dương thỏa mãn:$ab+bc+ac+5abc=8$.Chứng minh rằng:$15+ab+bc+ac\leq 2(a+b+c)^2$

 

b)Chứng minh rằng với mỗi số tự nhiên n>0 luôn tồn tại số tự nhiên A có n chữ số mà các chữ số của số A là 2 hoặc 5 sao A chia hết cho $2^{n}$

 

----HẾT ĐỀ 1----

 

Nhận xét,update lời giải, bình luận đề sau 2 tuần

 

Cuối  cùng mong ĐHV THCS có gì vi phạm thì ẩn hoặc sửa lại giúp mình

Mình xin giải câu bđt

Đặt $a+b+c=p ; ab+bc+ca=q, abc=r $

Ta có gt $<=> q + 5r =8 => r = \frac{8-q}{5} $

Theo bđt Schur bậc 3, ta có $p^3 -4pq + 9r \geq 0 <=> 5p^3 -20pq + 9(8-q) \geq 0$

$<=> q \leq \frac{5p^3+72}{20p+9} $

Mà ta cần chứng minh $15+q \geq 2p^2 $

Do đó, ta cần chứng minh $ \frac{5p^3+72}{20p+9} +15 \leq 2p^2 <=> 35p^3 + 18p^2-300p-207 \geq 0 <=> p \geq 3$(đúng)

Do đó bđt được chứng minh




#611170 Topic về Bất đẳng thức, cực trị THCS

Đã gửi bởi superpower on 26-01-2016 - 20:41 trong Bất đẳng thức và cực trị

Thế cái giả sử mà bạn nói tới nó có xẩy ra dấu "=" không... Chú ý tới dấu "=" đi 

Mk có lẽ là cách làm mình sai thật nhưng ..... chả biết nói sao h cả

Cách của bạn là hoàn toàn sai

Sai từ những kiến thức cơ bản

Và mình chỉ ra cái sai cho bạn

Còn cái dấu bằng thì nếu xét trường hợp có biến thôi, dễ mà




#611134 Topic về Bất đẳng thức, cực trị THCS

Đã gửi bởi superpower on 26-01-2016 - 17:10 trong Bất đẳng thức và cực trị

Xét theo từng vế 1 nhé

Vế trái ta cộng 3 vào:VT$=$ $\frac{x^2}{y^2+z^2}+1+\frac{y^2}{x^2+z^2}+1+\frac{z^2}{x^2+y^2}+1$

$=(x^2+y^2+z^2)(\frac{1}{y^2+z^2}+\frac{1}{x^2+z^2}+\frac{1}{x^2+y^2})$

Áp dụng BĐT cơ bản : $\frac{1}{a}+\frac{1}{b}+\frac{1}{c}\geq \frac{9}{a+b+c}$

Ta có VT $\geq (x^2+y^2+z^2).\frac{9}{2(x^2+y^2+z^2)}$$\doteq \frac{9}{x^2+y^2+z^2}$

Tương tự với VP cũng thêm 3 vào (2 vế cùng cộng vào 3)

VP$\geq \frac{9}{x+y+z}$

$\Rightarrow$ ĐPCM: $\frac{9}{x^2+y^2+z^2}\geq \frac{9}{x+y+z}$ luôn đúng $(x^2+y^2+z^2\geq x+y+z)$

Dấu "=" xẩy ra $x=y=z$

Bạn làm sai hoàn toàn

Khi bạn muốn chứng minh $a \geq b$

Bạn không thể so sánh $a \geq c ; b \geq d$ rồi so sánh $c,d$ được

Mình sẽ cho phản ví dụ

Giả sử; Ta cần chứng minh $ 5 \geq 10 $

Mặt khác $5 \geq 4 ; 10 \geq 3  ; 4 \geq 3$ thì bạn suy ra $5 \geq 10 $ à 




#605508 Topic về Bất đẳng thức, cực trị THCS

Đã gửi bởi superpower on 27-12-2015 - 15:17 trong Bất đẳng thức và cực trị

1, Cho 3 số thực a,b,c đều không nhỏ hơn $-\frac{3}{4}$, thỏa mãn $a+b+c= 1$ CMR:

                           $\frac{a}{a^{2}+1}+\frac{b}{b^{2}+1}+\frac{c}{c^{2}+1}\leq \frac{9}{10}$

Mình sẽ dùng phương pháp tiếp tuyến

Ta sẽ chứng minh 

$\frac{a}{a^2+1} \leq \frac{36a+3}{50} <=> 36(x-\frac{1}{3})^2 (x+\frac{3}{4} ) \geq 0 $ (Đúng)

Cộng theo vế, ta có điều phải chứng minh

Thực tế, ở bài toán này, có thể mở rộng ra $a,b,c$ là các số thực 




#617737 Tiếp sức bất đẳng thức

Đã gửi bởi superpower on 29-02-2016 - 21:32 trong Bất đẳng thức và cực trị

ai giúp bài này với

BÀI 91:

với a,b,c dương. a+b+c=10.

min:

$M= \frac{a^{2}-a}{9}+\frac{b^{2}-b}{5}+\frac{c^{2}-c}{3}$

Chứng minh bất đẳng thức phụ sau

$\frac{a^2-a}{9} \geq \frac{17}{27}a -\frac{100}{81} <=> (3a-10)^2 \geq 0 $

Do đó cộng các bất đẳng thức lại, ta có 

$M \geq \frac{17}{27} . 10 -\frac{100}{27} =\frac{70}{27} $




#613886 Tiếp sức bất đẳng thức

Đã gửi bởi superpower on 10-02-2016 - 10:13 trong Bất đẳng thức và cực trị

Bài dễ khai xuân đây : 
Bài 51. Cho $a,b,c \in \mathbb{R}$ và giả sử $a=max${$a,b,c$}. So sánh $b,c$ biết 
$(a-b)^5+(b-c)^5+(c-a)^5 \ge 0$

Cách khác cho bài 51

Do $a=max${$a,b,c$} $

Do đó $\exists  x,y: b=a-x ; c=a-y $

Thay vào, ta được 

$x^5 +(y-x)^5 -y^5 \geq 0 <=> (x^5-y^5) -( x-y)^5 \geq 0 <=> 5xy(x-y)(x^2 -xy+y^2) \geq 0 <=> x \geq y $

Do đó $b \leq c $




#620410 Tiếp sức bất đẳng thức

Đã gửi bởi superpower on 15-03-2016 - 19:57 trong Bất đẳng thức và cực trị

Bài 98:Cho $a,b,c$ là các số thực dương

C/m

$\sum \frac{a^4}{a^2+ab+b^2} \geq \frac{a^3+b^3+c^3}{a+b+c} $




#626707 Tìm x,y nguyên thỏa mãn đẳng thức: xy=xy+2

Đã gửi bởi superpower on 12-04-2016 - 09:00 trong Số học

Tìm x,y nguyên thỏa mãn đẳng thức: ​xy=xy+2

Xét $y$ âm, ta thấy vô lý

Do đó $y \geq 0 $

Xét $y=0 => 1 =2 $ vô lý

Xét $y \geq 1 $

Ta có $x^y -xy =2 => x(x^{y-1} -1) =2 $

TH1: $x=1$ vô lý

TH2: $x=2 => 2^{y-1} =2 => y=2 $

TH3: $x=-1$ vô lý

TH4: $x=-2 => (-2)^{y-1} =2$ vô lý

Vậy $x=2; y =2 $ là nghiệm nguyên duy nhất 




#591621 Tìm n để n! là số chính phương

Đã gửi bởi superpower on 01-10-2015 - 22:27 trong Số học

n=1

n=1 xét riêng được mà. 1 thì đặc biệt quá. Lời giải trên kia còn nhiều chỗ hở




#591613 Tìm n để n! là số chính phương

Đã gửi bởi superpower on 01-10-2015 - 22:15 trong Số học

Tìm tất cả các số tự nhiên n sao cho n! là số chính phương.

Theo mình nghĩ là không tồn tại n nhé

Vì lấy số nguyên tố gần nhất với n

Thì không còn số nào chia hết cho p hết ( Vì tích của n số tự nhiên đầu tiên )

Nên không thể là số chính phương. 




#591738 Tìm n để n! là số chính phương

Đã gửi bởi superpower on 02-10-2015 - 21:05 trong Số học

Vậy thì ta phải cm $p<n<p^2$ trong đó p là ước nguyên tố lớn nhất của n. Điều này không hiển nhiên đâu nhé.

do n! mà. Nên cũng dễ hiểu. Mà bây giờ tìm cách chứng minh cho thuyết phục




#634693 Tìm GTLN $\frac{x}{1+x^2} + \frac{y...

Đã gửi bởi superpower on 22-05-2016 - 14:40 trong Bất đẳng thức và cực trị

Cho x,y,z > 0 và x+y+z=1

Tìm GTLN P=$\frac{x}{1+x^2} + \frac{y}{1+y^2} + \frac{z}{1+z^2}$

 

Ta có

$\frac{x}{x^2+1}-\frac{18x}{25}-\frac{3}{50}=\frac{-(4x+3)(3x-1)^2}{50(x^2+1)}\leq 0$

Suy ra

$\frac{x}{x^2+1}\leq\frac{18x}{25}+\frac{3}{50}$

Tương tự, rồi cộng lại có

$\sum\frac{x}{x^2+1}\leq\sum(\frac{18x}{25}+\frac{3}{50})=\frac{18}{25}\sum x+\frac{9}{50}=\frac{9}{10}$

Dấu $"="$ xảy ra khi

$x=y=z=\frac{1}{3}$

Vậy ...

Bài này có thể mở rộng cho $x,y,z \in R $

Chứng minh như sau

Không mất tính tổng quát giả sử $x \geq y \geq z $

Nếu $z  \geq \frac{-3}{4} $ thì như lời giải trên ta có điều phải chứng minh

Xét TH $z \leq \frac{-3}{4} $ thì

$\frac{z}{z^2+1} \leq \frac{-12}{25} $ Do $z \leq \frac{-3}{4} $ và $z^2 +1 \geq \frac{25}{16} $

Ta cũng có $\frac{x}{x^2+1} \leq \frac{1}{2} $

Tương tự cho $y$

Do đó cộng lại, ta được $P \leq \frac{-12}{25} +1 < \frac{9}{10} $




#605146 Thắc mắc về dấu bằng trong một bài toán bất đẳng thức

Đã gửi bởi superpower on 25-12-2015 - 09:43 trong Bất đẳng thức - Cực trị

Trong cuốn "Những viên kim cương trong bất đẳng thức toán học" của thầy Trần Phương, bài số 6 trang 269 có hai trường hợp dấu $"="$ xảy ra đó là $a=b=c$ hoặc $a=b;c=0$ và các hoán vị. Cả hai trường hợp này khi thay vào đều thỏa mãn. Nhưng khi áp dụng $q^3 \geq 27r^2$ thì dấu bằng chỉ xảy ra khi $a=b=c$. Vậy tại sao tồn tại 2 dấu bằng?

Nếu mà dấu bằng dạng này, bạn coi lại thử đi

có thể là $r.q^3 \geq 27r^3$